IMO Shortlist 2000

2017. 12. 31. 13:03수학 문풀/기타

"AOPS 문제들이 너무 체계적이지 않다"라고 친구에게 불평했더니 IMO 쇼트를 풀어보라는 답변이 돌아왔다. 그 친구의 말을 믿어도 될런지 모르겠지만... 일단 풀거나 Give up한 순서대로 포스팅해보자.


N2. (solved, 171231)

Find all natural number \(n\) s.t. \(\tau (n)^3 = 4n\).

전형적인 수론함수 노가다 문제이다. 내가 잘 푸는 몇 안되는 유형...

일단 \(n\)을 다음과 같이 소인수분해할 수 있음이 자명하다.


$$ n = 2^{3b+1} \cdot p_{1}^{3a_1}p_{2}^{3a_2} \cdots p_{t}^{3a_{t}}$$


이 때 \((\frac{\tau(n)^3}{4n})^{\frac{1}{3}} = \frac{3b+2}{2^{b+1}} \cdot \prod_{i=1}^{t} \frac{3a_{i}+1}{p_i^{a_i}} = 1\)의 해를 찾고 싶다.


이 때 임의의 \(a\)와 임의의 홀수 소수 \(p\)에 대해 \(\frac{3a+1}{p^a} \le \frac{4}{3}\)가 성립함을 쉽게 증명할 수 있으므로, \(\frac{3a+1}{p^a} < \frac{3}{4}\)인 \(a,p\)는 사용할 수 없다.


이 때 \(p > 7\)인 경우 \(\frac{3a+1}{p^a} < \frac{4}{7} < \frac{3}{4}\)이므로 불가능하다.


효율적인 counting을 위해 \(b\)를 생각해 보자. \((f(b) := \frac{3b+2}{2^{b+1}}\)


\(b = 0\) : \(f(b) = 1\)이므로 자체로 solution이 될 수 있다. \(n=2\) 의 해를 찾았다. \(p = 3,5\)에 대해서 \(\frac{3a+1}{p^a} = 1\)인 \(a\)가 존재하지 않으므로 추가적인 해는 없다.


\(b = 1\) : \(f(b) = \frac{5}{4}\)이므로 분모에 5를 추가해야 한다. \(p = 5, a = 1\)인 경우가 가능하고, 이 때 \(n = 2000\)의 해를 찾았다. 마찬가지로 다른 해는 없다.


\(b = 2\) : \(f(b) = 1\)이다. 이 자체로 해이고, \(n = 128\)이다. 추가적인 해는 마찬가지로 없다.


\(b > 3\) : \(f(b) \le \frac{7}{16} < \frac{3}{4}\)이므로 더 이상의 해는 없다.


따라서 가능한 \(n\)은  \(2, 128, 2000\)뿐이다.


A1. (Gived Up, 180101)

For all positive real number \(a,b,c\) satisfy \(abc = 1\), 

Prove that \((a-1+\frac{1}{b})(b-1+\frac{1}{c})(c-1+\frac{1}{a}) \le 1\).

치환 아이디어까진 좋았는데 계산병이 도져서....


양수 \(x,y,z\)에 대해 \(a = \frac{x}{y}, b = \frac{y}{z}, c = \frac{z}{x}\)라고 두면, 주어진 명제는 다음과 같이 바뀐다.


$$ (x-y+z)(y-z+x)(z-x+y) \le xyz $$


이 때 \(x-y+z = p, y-z+x = q, z-x+y = r\)이라고 두면 \(p+q, q+r, r+p > 0\)이므로 \(p,q,r\) 중 최대 하나만 음수이다.


또한 주어진 부등식은 다음과 같이 바뀐다.


$$ 8pqr \le (p+q)(q+r)(r+p) $$


이 때 \(p,q,r\)중 하나가 음수인 경우 좌변은 음수, 우변은 양수이므로 성립. 셋 다 양수인 경우는 \(\text{AM-GM}\)에 의해 성립한다. \(\blacksquare\)

'수학 문풀 > 기타' 카테고리의 다른 글

Putnam 2017 풀이 - 풀리는 것만  (0) 2018.11.30
2018 대수경 2분야 5번 풀이  (0) 2018.11.24
2017 Benelux MO 풀이  (0) 2018.09.29
170128 함수방정식  (0) 2017.10.28
Titu - 104 정수론 풀이 #003  (0) 2017.07.14